How can I use direct integration to solve for the convolution of two signals?

Click For Summary
To solve the convolution of two signals using direct integration, the convolution integral must be set up correctly with the given functions x(t) and h(t). The user is struggling with the integration process, particularly with the inequalities that arise from the unit step functions involved. Understanding the role of the unit step function u(t) is crucial for interpreting the limits of integration. The discussion highlights the complexity of the problem compared to simpler graphical methods, emphasizing the need for clarity in handling the inequalities. Direct integration can be challenging, but breaking down the problem into manageable parts may help simplify the process.
Vanush
Messages
24
Reaction score
0
Hey guys, I'm having trouble doing ct convolution

i'm trying to convolve two signals together ie, the input x(t) and the impulse response h(t). basically, knowing the impulse response of an LTI system, you can find out the response y(t) to any arbitrary input x(t) using the convolution integral.

in my problem

x(t) = t * ( u(t) - 2*( u(t - 1) + u(t - 2)),
h(t) = u(-t) - u(-t + 1)

So i had a look at examples of calculating the integral using the graphical method, and i get a triangle signal as y(t), convolving the signals above. however, the question wants me to do it using direct integration. I have no idea how to do this! Anyone have any ideas?
 
Physics news on Phys.org
What is u(t)?
 
Unit step function
 
He wasn't asking you what that notation meant, but how you should think of interpreting what it means when integrating it.
 
THat's the thing, when I put it into the integral I have to work out a bunch of inequalities that really makes my head spin. This question is so much harder than the example :(
 
Question: A clock's minute hand has length 4 and its hour hand has length 3. What is the distance between the tips at the moment when it is increasing most rapidly?(Putnam Exam Question) Answer: Making assumption that both the hands moves at constant angular velocities, the answer is ## \sqrt{7} .## But don't you think this assumption is somewhat doubtful and wrong?

Similar threads

  • · Replies 1 ·
Replies
1
Views
1K
  • · Replies 4 ·
Replies
4
Views
3K
  • · Replies 6 ·
Replies
6
Views
2K
  • · Replies 3 ·
Replies
3
Views
2K
  • · Replies 6 ·
Replies
6
Views
1K
  • · Replies 3 ·
Replies
3
Views
2K
  • · Replies 3 ·
Replies
3
Views
2K
  • · Replies 3 ·
Replies
3
Views
1K
  • · Replies 2 ·
Replies
2
Views
2K
  • · Replies 1 ·
Replies
1
Views
2K